Lorentz transform of a wavepacket

In summary, the problem is asking for a function that is invariant under Lorentz transformation, but the explicit functional form of this function is not known. It may be possible to get this function by transforming its arguments, depending on the type of field involved.
  • #1
bjnartowt
284
3

Homework Statement



see attached .pdf. all parts of problem statement are italicized.

Homework Equations



see attached .pdf

The Attempt at a Solution



see attached .pdf


Actually: my question is pretty qualitative. You can look at everything I've done with this problem so far. However, the problem is asking for psi_PRIME, that is, the function in the new frame of reference. As you can see, I've proved the wave equation is invariant under Lorentz transform. Also, I don't think it's a mean feat to transform the x and t arguments of psi to make psi(x_PRIME, t_PRIME). However, what exactly *is* psi_PRIME, beyond the wave in the new frame of reference? I'm not sure how to "get" psi_PRIME. Do I "get" psi_PRIME when I transform its arguments? Then I think psi = psi_PRIME, because I'd be plugging transformed (primed) variables into the same ol' function...
 

Attachments

  • 578 - pr 35 - lorentz transform of wavepacket.pdf
    33.2 KB · Views: 246
Physics news on Phys.org
  • #2
[tex]\psi'(x', t') = A(\Lambda) \psi(x(x',t'), t(x',t')) A^{-1}(\Lambda)[/tex] where A(Λ) gives you the group theoretical factor which depends on the type of field you have (more exactly, its spin). If ψ is a scalar, then A = 1.
 
  • #3
Ok, thank you. So you're saying that if psi was a vector, the explicit functional form of psi would transform, and not just the variables x --> xprime and t --> tprime? Because my psi is a scalar (it's a one-dimensional wavepacket).
 
  • #4
Exactly. If your ψ is a scalar, you only transform its arguments (x, t). If it would be a spinor, or a vector, or ..., you would have to multiply by some matrix A which depends on your Lorentz transformation Λ. This is very well explained in Peskin and Schroeder's book "An Introduction to QFT", if you have a library near you.

(BTW: Peskin & Schroeder is one of the most readable books on QFT I have found so far, if you plan to dig into QFT, get this one.)
 
  • #5


I can provide a response to your question about the Lorentz transform of a wavepacket. The Lorentz transform is a mathematical tool that allows us to relate physical quantities in one frame of reference to those in another frame of reference that is moving at a constant velocity relative to the first frame. In this case, we are interested in the Lorentz transform of a wavepacket, which is a localized disturbance in a medium that propagates through space and time.

The Lorentz transform of a wavepacket can be obtained by applying the Lorentz transform to the coordinates of each point in the wavepacket. This means that the position and time coordinates of each point in the wavepacket are transformed according to the Lorentz transform equations. This will result in a new wavepacket in the new frame of reference, with transformed coordinates and possibly different shape and size.

It is important to note that the Lorentz transform preserves the wave equation, which describes the behavior of the wavepacket in both frames of reference. This means that the wavepacket in the new frame of reference will still satisfy the wave equation, just with transformed coordinates. Therefore, the wavepacket in the new frame of reference, psi_PRIME, is essentially the same function as the original wavepacket, psi, but with transformed coordinates.

In summary, to "get" psi_PRIME, you would need to apply the Lorentz transform to the coordinates of the wavepacket, using the Lorentz transform equations. This will result in a new wavepacket in the new frame of reference, with transformed coordinates and still satisfying the wave equation. I hope this helps clarify the concept of the Lorentz transform of a wavepacket.
 

Related to Lorentz transform of a wavepacket

1. What is the Lorentz transform of a wavepacket?

The Lorentz transform of a wavepacket is a mathematical formula that describes how a wavepacket, which is a localized disturbance or wave in a physical system, changes when viewed from different frames of reference in special relativity. It takes into account the effects of time dilation and length contraction.

2. How is the Lorentz transform different from the Galilean transform?

The Lorentz transform is different from the Galilean transform in that it takes into account the principles of special relativity, such as the constancy of the speed of light and the relativity of simultaneity. The Galilean transform is based on classical mechanics and does not consider these principles.

3. What is the significance of the Lorentz transform in physics?

The Lorentz transform is significant in physics because it allows us to accurately describe the behavior of physical systems at high speeds and in different frames of reference. It is a key component of Einstein's theory of special relativity and has been confirmed through numerous experiments and observations.

4. How is the Lorentz transform applied in practical situations?

The Lorentz transform is applied in practical situations, such as in particle accelerators, where particles are accelerated to high speeds. It is also used in GPS technology, as the satellites in orbit must take into account the effects of special relativity to accurately calculate position and time measurements on Earth.

5. Are there any limitations to the Lorentz transform?

While the Lorentz transform accurately describes the behavior of physical systems in special relativity, it does have limitations. It does not apply to systems with acceleration or gravitational effects, which require the more comprehensive theory of general relativity. Additionally, it is only applicable to systems moving at constant velocities in a straight line.

Similar threads

  • Advanced Physics Homework Help
Replies
4
Views
2K
  • Advanced Physics Homework Help
Replies
10
Views
386
  • Special and General Relativity
3
Replies
101
Views
3K
  • Advanced Physics Homework Help
Replies
1
Views
805
  • Special and General Relativity
Replies
10
Views
614
  • Advanced Physics Homework Help
Replies
3
Views
1K
  • Advanced Physics Homework Help
Replies
11
Views
2K
  • Advanced Physics Homework Help
Replies
13
Views
2K
Replies
4
Views
2K
  • Special and General Relativity
Replies
5
Views
965
Back
Top